2
$\begingroup$

In Kerson Huang's Quantum Field Theory 2nd, p 278, the author introduces "cumulant expansion" from

$$ \ln \langle e^x \rangle = \ln \sum_{n=0}^\infty \frac{\langle x^n \rangle}{n!} = \langle x \rangle + \frac{1}{2} [ \langle x^2 \rangle - \langle x \rangle^2 ] + \frac{1}{6} [ \langle x^3 \rangle - 3 \langle x \rangle \langle x^2 \rangle + \langle x\rangle^3 ] + \cdots (14.112) $$

However, if I start from $$\ln \langle e^x \rangle = \ln( 1 + \langle x \rangle + \frac{\langle x^2 \rangle}{2!} + \frac{ \langle x^3 \rangle}{3!} + \cdots) $$

use the Taylor expansion $\ln (1+x) = x - \frac{x^2}{2} + \frac{x^3}{3} + \cdots$ I got $$ \langle x \rangle + \frac{\langle x^2\rangle}{2!} +\frac{ \langle x^3 \rangle}{3!} + \cdots - \frac{1}{2} \left(\langle x \rangle + \frac{\langle x^2 \rangle }{2!}+ \cdots \right)^2 + \frac{1}{3} \left( \langle x \rangle + \cdots \right)^3 $$

I have the terms at $x$ and $x^2$ correct. For the cubic, I have $$ \frac{1}{6} \left( \langle x^3 \rangle - 3 \langle x\rangle \langle x^2 \rangle + 2 \langle x \rangle ^3 \right)$$ Why the $\langle x \rangle^3$ differs with the prefactor?

$\endgroup$
5
  • 1
    $\begingroup$ must be a typo on its end. Indeed, the kurtosis $\kappa_3=\langle (x-\langle x \rangle)^3\rangle$ expands to what you give. $\endgroup$ Commented Oct 3 at 8:46
  • $\begingroup$ Sometimes textbooks contain typos. You either trust yourself or check in another book. Kudos for finding a typo (+1)... but as a question on SE this is not very useful. $\endgroup$ Commented Oct 3 at 12:07
  • $\begingroup$ I may have Kerson's errata for the book in my office, and if I can find it I will check this point. $\endgroup$ Commented Oct 3 at 13:50
  • $\begingroup$ Could you recommend a book for cumulant expansion especially quantum aspects? I searched a bit but still confusing. Thank you very much @RogerV. $\endgroup$ Commented Oct 3 at 18:26
  • $\begingroup$ Cumulant expansion is a standard statistical tool, not specific to QFT. You may want to start with WIkipedia: fr.wikipedia.org/wiki/… There is also a couple of references to books on statistics in the end. $\endgroup$ Commented Oct 3 at 19:46

0

Your Answer

By clicking “Post Your Answer”, you agree to our terms of service and acknowledge you have read our privacy policy.

Start asking to get answers

Find the answer to your question by asking.

Ask question

Explore related questions

See similar questions with these tags.